Question on nonlinear dynamics

Click For Summary
The discussion revolves around the existence of a stable T-periodic solution for the system x' + x = F(t), where F is a smooth, T-periodic function. One participant believes that a T-periodic solution must exist and proposes to eliminate the time parameter to analyze the system as a 2D system. They plan to use a Poincaré map to demonstrate the periodicity of the solution. Another participant suggests considering a Fourier expansion as an alternative approach. The conversation highlights the need for further exploration of the proposed methods to confirm the existence of the periodic solution.
JuanYsimura
Messages
5
Reaction score
0

Homework Statement



Consider the system x' + x = F(t), where F is smooth, T-periodic function. IS it true that the system necessarily has a stable T-periodic solution x(t)? If so, Prove it; If not, find an F that provides a counterexample.

Homework Equations



x' + x = F(t)


The Attempt at a Solution



So, I think this system necessarily have a T-periodic solution x(t). To prove it, I let y = t so i can eliminate the time parameter from the equation so then I get a 2D system

x' = F(y) - x
y' = 1

SO, then I plan to find a poincare map so that showing that x(t) is T-periodic.

MY question is, before I go further, is this a good approach to the problem?

thanks.
 
Physics news on Phys.org
could you consider a Fourier expansion?
 
Question: A clock's minute hand has length 4 and its hour hand has length 3. What is the distance between the tips at the moment when it is increasing most rapidly?(Putnam Exam Question) Answer: Making assumption that both the hands moves at constant angular velocities, the answer is ## \sqrt{7} .## But don't you think this assumption is somewhat doubtful and wrong?

Similar threads

Replies
6
Views
2K
Replies
6
Views
1K
  • · Replies 11 ·
Replies
11
Views
3K
Replies
3
Views
2K
  • · Replies 1 ·
Replies
1
Views
2K
  • · Replies 1 ·
Replies
1
Views
2K
  • · Replies 3 ·
Replies
3
Views
2K
  • · Replies 3 ·
Replies
3
Views
2K
Replies
4
Views
2K
Replies
8
Views
2K